9. 5 people each pull with 200 N each on an object. What is the net force on the object?

Answers

Answer 1

Answer:

1000 N

Explanation:

The net force is sum of all forces when considering in a one-dimensional motion. Since each people pull the object 200 N then that means 5 people together would pull 5 × 200 N = 1000 N.

Therefore, the net force is 1000 N


Related Questions

The air inside a hot-air balloon has an average temperature of 78.0 ∘C. The outside air has a temperature of 21.8 ∘C. What is the ratio of the density of air in the balloon to the density of air in the surrounding atmosphere?

Answers

The ratio of the density of air in the balloon to the density of air in the surrounding atmosphere is approximately 1.186.

How to calculate the ratio of the densities of air in the balloon and the surrounding atmosphere?

To calculate the ratio of the density of air in the balloon to the density of air in the surrounding atmosphere, we can use the ideal gas law.

The ideal gas law is given by:

PV = nRT

where P is the pressure, V is the volume, n is the number of moles of gas, R is the gas constant, and T is the temperature in Kelvin.

The density of air (ρ) is related to the pressure, volume, and temperature by the equation:

ρ = (P / RT)

We can use this equation to compare the densities of air in the balloon and the surrounding atmosphere.

Let's denote the density of air inside the balloon as ρ_balloon and the density of air in the surrounding atmosphere as ρ_atmosphere.

The ratio of the densities can be expressed as:

Ratio = ρ_balloon / ρ_atmosphere

Using the ideal gas law equation, we can rewrite the ratio as:

Ratio = (P_balloon / RT_balloon) / (P_atmosphere / RT_atmosphere)

Since the pressure and gas constant are the same for both the balloon and the atmosphere, they cancel out in the ratio expression.

The temperature needs to be converted to Kelvin:

T_balloon = 78.0 °C + 273.15 = 351.15 K

T_atmosphere = 21.8 °C + 273.15 = 295.95 K

Now, we can calculate the ratio:

Ratio = (T_balloon / T_atmosphere)

Substituting the given values:

Ratio = 351.15 K / 295.95 K

Ratio ≈ 1.186

Therefore, the ratio of the density of air in the balloon to the density of air in the surrounding atmosphere is approximately 1.186.

Learn more about the surrounding atmosphere

brainly.com/question/30276789

#SPJ11

determine the entropy of the sum that is obtained when a pair of fair dice are rolled.

Answers

The entropy of the sum obtained when a pair of fair dice are rolled can be determined by calculating the probability distribution of the sum and using it to compute the entropy.

When two dice are rolled, there are 36 possible outcomes, each with equal probability.

The sum of the two dice ranges from 2 to 12, with different numbers of possible outcomes for each sum.

The probability distribution for the sum is a discrete probability distribution with unequal probabilities.

Using this probability distribution, the entropy of the sum can be calculated using the formula for entropy.

Moreover, performing certain calculations also gives us the value of the entropy for the sum obtained when rolling a pair of fair dice.

Read more about Entropy.

https://brainly.com/question/20166134

#SPJ11

light from a he-ne laser (λ=632.8nm) strikes a pair of slits at normal incidence, forming a double-slit interference pattern on a screen located 1.40 m from the slits The figure(Figure 1) shows the interference pattern observed on the screen. What is the slit separation? d=____um

Answers

The slit separation is approximately 0.34 μm.
From the interference pattern observed on the screen, we can see that there are bright fringes (maxima) and dark fringes (minima) of intensity. The distance between adjacent bright fringes (or dark fringes) is given by the equation:

y = (λL) / d

where y is the distance from the central maximum to the nth bright fringe (or dark fringe), λ is the wavelength of the light, L is the distance between the slits and the screen, and d is the slit separation.

Using the given values, we can find the distance between adjacent bright fringes:

y = (632.8 nm) * (1.40 m) / d

The first bright fringe is located at y = 0.9 mm, and the second bright fringe is located at y = 1.8 mm. Therefore, the distance between adjacent bright fringes is:

Δy = 0.9 mm - 0 mm = 0.9 mm

We can use this value to find the slit separation:

Δy = (λL) / d

d = (λL) / Δy

Substituting the given values, we get:

d = (632.8 nm) * (1.40 m) / (0.9 mm)

d ≈ 0.34 μm

learn more about slit separation

https://brainly.com/question/17167388

#SPJ11

complete the following nuclear reaction: 73li 11h→42he ?

Answers

The complete nuclear reaction is: 73Li + 11H -> 42He + 9Be.

Here, the sum of the mass numbers and atomic numbers on both sides of the equation must be equal.

On the left-hand side of the equation, we have 7 protons and 3 neutrons from 73Li, and 1 proton from 11H. Thus, the total mass number is 7 + 3 + 1 = 11, and the total atomic number is 3 + 1 = 4.

On the right-hand side of the equation, we have 2 protons and 2 neutrons from 42He. Therefore, the missing product must have a mass number of 9 (11 - 2) and an atomic number of 2 (4 - 2). The only isotope that fits this description is 9Be, which has 4 protons and 5 neutrons.

For more such questions on nuclear reaction:

https://brainly.com/question/12786977

#SPJ11

The complete nuclear reaction is: 73Li + 11H -> 42He + 9Be.

Here, the sum of the mass numbers and atomic numbers on both sides of the equation must be equal.

On the left-hand side of the equation, we have 7 protons and 3 neutrons from 73Li, and 1 proton from 11H. Thus, the total mass number is 7 + 3 + 1 = 11, and the total atomic number is 3 + 1 = 4.

On the right-hand side of the equation, we have 2 protons and 2 neutrons from 42He. Therefore, the missing product must have a mass number of 9 (11 - 2) and an atomic number of 2 (4 - 2). The only isotope that fits this description is 9Be, which has 4 protons and 5 neutrons.

Visit to know more about Nuclear reaction:-
brainly.com/question/12786977

#SPJ11

A sample of 0.351 mol of a metal M
reacts completely with excess fluorine to form 27.4 g of M
F
2
. Identify the metal M
.

Answers

The metal M in the given reaction is likely Calcium (Ca).

To identify the metal M, we need to determine its atomic mass and the atomic mass of M can be calculated using molar mass of MF₂.

The molar mass of MF₂ can be calculated as:

Molar mass of MF₂ = Molar mass of M + 2 × Molar mass of F

                                = M + 2 × 18.998 g/mol

                                = M + 37.996 g/mol

Given, mass of MF₂ formed = 27.4 g

We know that 0.351 mol of M reacts with excess fluorine to form 27.4 g of MF₂. Therefore, we can use the molar mass of MF₂ and the mass of MF₂ formed to find the moles of MF₂ as;

27.4 g / (M + 37.996 g/mol) = 0.351 mol

                         M + 37.996 = 27.4 / 0.351

Solving for M, we get:

M = (27.4 / 0.351) - 37.996

   = 40.07 g/mol

Therefore, the metal M has an atomic mass of 40.07 g/mol. Looking at the periodic table, we see that the only metal with a similar atomic mass is Ca (Calcium).

Therefore, the metal M is likely Calcium (Ca).

Learn more about molar mass here

brainly.com/question/30640134

#SPJ4

A firm in monopolistic competition faces a demand function equal to
P=200-2Q
and a cost function equal to
C(Q)=10+4Q
The profit max level of output equals ____ units
The answer is 49 but how did you get it? Can you please go step by step and write legibly.

Answers

To find the profit-maximizing energy level of output for a firm in monopolistic competition, we need to use the following formula: MC = MR, Where MC is the firm's marginal cost and MR is the firm's marginal revenue.

The profit-maximizing level of output for the firm is 49 units. To find the profit at this level of output, we plug Q = 49 into the demand and cost functions:
P = 200 - 2(49) = 102
C(Q) = 10 + 4(49) = 206
Profit = Total revenue - Total cost
Profit = P * Q - C(Q)
Profit = 102 * 49 - 206
Profit = 4,988

In this case, the profit-maximizing level of output is 49 units. This is because, at this level of output, the marginal profit is zero, meaning any additional units produced would not increase profit further.

To know more about matter visit:

https://brainly.com/question/1932868

#SPJ11

Mexico was able to gain its independence from Spain when which group switched sides to the cause of independence

Answers

Mexico was able to gain its independence from Spain when the Criollos (Mexican-born Spaniards) switched sides to the cause of independence.

The Criollos, who were previously loyal to the Spanish crown, became disillusioned with Spanish rule and were influenced by the ideals of the American and French revolutions. They recognized the need for political and economic autonomy, leading them to support the Mexican independence movement. Their defection significantly bolstered the strength and legitimacy of the movement, providing crucial leadership, resources, and military support. The Criollos played a vital role in organizing and leading the struggle for independence, ultimately leading to Mexico's successful break from Spanish colonial rule in 1821.

learn more about Mexico  here:

https://brainly.com/question/12173994

#SPJ11

solve the emp to find the hicksian demand function, h (p, u)

Answers

The Hicksian demand function h(p, u) represents the optimal consumption bundle that minimizes expenditure given prices p and a fixed utility level u.

To find the Hicksian demand function, h(p, u), follow these steps:
1. Determine the utility function, which reflects consumers' preferences.
2. Calculate the expenditure function by minimizing the cost of achieving utility level u, given prices p.
3. Derive the Marshallian demand function, which shows the optimal consumption bundle given prices p and income.
4. Apply the Shepard's lemma to the expenditure function to obtain the Hicksian demand function, h(p, u), which shows the consumption bundle that minimizes expenditure while maintaining a constant utility level u.

In this process, you will obtain the Hicksian demand function, which is a key concept in consumer theory and represents the optimal consumption choices to minimize expenditure given prices and a fixed utility level.

Learn more about optimal consumption here:

https://brainly.com/question/29997674

#SPJ11

Two very long, parallel wires are separated by d = 0.065 m. The first wire carries a current of I1 = 0.65 A. The second wire carries a current of I2 = 0.35 A.1) Express the magnitude of the force between the wires per unit length, f, in terms of I1, I2, and d.2)Calculate the numerical value of f in N/m.3)Is the force repulsive or attractive?4) Express the minimal work per unit length needed to separate the two wires from d to 2d.5)Calculate the numerical value of w in J/m.

Answers

1) Express the magnitude of the force between the wires per unit length, f, in terms of I1, I2: f = (μ0/4π) * (I1 * I2 / d),

2) Calculate the numerical value of f in N/m: 9.86 x 10^-5 N/m

3) The force is repulsive.

4) Express the minimal work per unit length needed to separate the two wires from d to 2d: 1.15×10⁻⁸ J/m

5) The numerical value of w in J/m is: 6.4 x 10^-6 J/m.

Explanation to above written short answers are given below,

1. The magnitude of the force between the wires per unit length, f, in terms of I1, I2, and d can be expressed by the equation
f = (μ0/4π) * (I1 * I2 / d),
where μ0 is the permeability of free space.

2. Substituting the given values, we get
f = (4π x 10^-7 N/A^2) * (0.65 A * 0.35 A / 0.065 m) = 9.86 x 10^-5 N/m.

3. The force between the wires is attractive since the currents are in opposite directions.

4. To separate the two wires from d to 2d, we need to do work against the magnetic field produced by the current-carrying wires. The work required per unit length is given by:
W/L = μ₀I₁I₂ln(2)
where μ₀ is the permeability of free space,
I₁ and I₂ are the currents in the wires, and
ln(2) is the natural logarithm of 2.

Substituting the given values, we get:
W/L = (4π×10⁻⁷ T·m/A) × (0.65 A) × (0.35 A) × ln(2) = 1.15×10⁻⁸ J/m

5. Substituting the value of f from above, we get
W = ∫(9.86 x 10^-5 N/m)dx from d to 2d.

Solving this integral gives us
W = 9.86 x 10^-5 N/m * (2d - d) = 9.86 x 10^-5 N/m * d = 6.4 x 10^-6 J/m.

To know more about "magnetic field" refer here:

https://brainly.com/question/12578759#

#SPJ11

A positive charge 1.1X10-11 C is located 10-2 m away from a negative charge of the same magnitude. Point P is exactly half way between them --what is the E field at point P? a. 103 N/C b. 2X103 N/C c. 4X103 N/C d. 8X103 N/C

Answers

The electric field at point P is 4 X [tex]10^3[/tex] N/C (option c), due to the cancellation of equal and opposite charges.

In this situation, a positive charge of 1.1 X [tex]10^{-11[/tex] C and a negative charge of the same magnitude are placed [tex]10^{-2[/tex] m apart. Point P is located exactly halfway between them.

Since the charges are equal and opposite, their electric fields at point P will be equal in magnitude but opposite in direction. As a result, the electric fields will partially cancel each other out.

The net electric field at point P can be calculated using the superposition principle, and the final result is 4 X [tex]10^3[/tex] N/C. Thus, the correct choice is (c).

For more such questions on electric, click on:

https://brainly.com/question/1100341

#SPJ11

The E field at point P is [tex]4 * 10^3 N/C[/tex]. The correct answer is C.

To find the electric field at point P, we need to consider the contributions from both charges. Since the charges have the same magnitude and are equidistant from point P, the electric fields they produce will have the same magnitude but opposite directions.

The electric field due to a point charge can be calculated using the equation:

[tex]E = k * (|q| / r^2)[/tex]

where E is the electric field, k is the Coulomb's constant [tex](9 * 10^9 N m^2/C^2)[/tex], |q| is the magnitude of the charge, and r is the distance from the charge.

In this case, the distance between each charge and point P is [tex]10^(-2)/2 = 5 * 10^(-3) m.[/tex]

The electric field due to each charge at point P is:

[tex]E1 = k * (|q| / r^2) = (9 * 10^9 N m^2/C^2) * (1.1 * 10^{(-11)} C / (5 * 10^{(-3)} m)^2)[/tex]

[tex]E2 = k * (|q| / r^2) = (9 * 10^9 N m^2/C^2) * (1.1 * 10^{(-11)} C / (5 * 10^{(-3)} m)^2)[/tex]

Since the electric fields have opposite directions, the net electric field at point P is the vector sum of E1 and E2.

[tex]|E1 + E2| = |E1| - |E2|[/tex]

Substituting the values:

[tex]|E1 + E2| = (9 * 10^9 N m^2/C^2) * (1.1 * 10^{(-11)} C / (5 x 10^{(-3)} m)^2) - (9 * 10^9 N m^2/C^2) * (1.1 * 10^{(-11)} C / (5 x 10^{(-3)} m)^2)[/tex]

Calculating the above expression, we find that [tex]|E1 + E2|[/tex] is approximately [tex]4 * 10^3 N/C.[/tex]

Therefore, the correct answer is c) [tex]4 * 10^3 N/C.[/tex]

To learn more about E field from the given link

https://brainly.com/question/19878202

#SPJ4

can the radial velocity method only be used with white dwarf stars
True or False

Answers

The radial velocity method can only be used with white dwarf stars True

The given statement " can the radial velocity method only be used with white dwarf stars" is false.

The radial velocity method is a technique used in astronomy to detect exoplanets by measuring the Doppler shift of the host star's spectral lines as the star wobbles due to the gravitational influence of the orbiting planet.

This method can be the used with various types of stars, not just white dwarf stars. In fact, the radial velocity method has been used to discover thousands of exoplanets orbiting a wide variety of stars, including main-sequence stars, giant stars, and even some brown dwarfs.

For such more question on velocity:

https://brainly.com/question/80295

#SPJ11

A ladder 6.10 m long leans against a wall inside a spaceship. From the point of view of a person on the ship, the base of the ladder is 2.70 m from the wall, and the top of the ladder is 5.47 m above the floor. The spaceship moves past the Earth with a speed of 0.83c in a direction parallel to the floor of the ship. What is the length of the ladder as seen by an observer on Earth?

Answers

The length of the ladder is approximately 3.40 meters.

To find the length of the ladder as seen by an observer on Earth, we need to consider the Lorentz transformation, which accounts for the length contraction due to the relativistic effect at high speeds.

The terms involved are the proper length (L₀), the length observed by the Earth observer (L), and the spaceship's speed (v) as a fraction of the speed of light (c).

The proper length (L₀) is the length of the ladder as measured by the person inside the spaceship, which is 6.10 m. The spaceship is moving with a speed of 0.83c.

Using the length contraction formula, L = L₀ * √(1 - v²/c²), we can find the length of the ladder observed by the Earth observer:

L = 6.10 m * √(1 - (0.83c)²/c²)
L ≈ 6.10 m * √(1 - 0.6889)
L ≈ 6.10 m * √(0.3111)
L ≈ 6.10 m * 0.5576
L ≈ 3.40 m

As seen by an observer on Earth, the length of the ladder is approximately 3.40 meters.

To learn more about earth, refer below:

https://brainly.com/question/31064851

#SPJ11

is the wavelength of the fundamental standing wave in a tube open at both ends greater than equal to or less than the wavenlegth for the fundamental wave in a tube open at just one end

Answers

The wavelength of the fundamental standing wave in a tube open at both ends is greater than the wavelength for the fundamental wave in a tube open at just one end.

This is because in a tube open at both ends, the waves reflect back and forth between the two ends and interference causes nodes (points of zero displacement) to occur at both ends. In a tube open at just one end, only one end is fixed and the waves reflect back from the open end, causing a node to occur at the fixed end and an antinode (point of maximum displacement) to occur at the open end. Therefore, the wavelength in a tube open at both ends is twice the length of the tube, while the wavelength in a tube open at just one end is four times the length of the tube.

The wavelength of the fundamental standing wave in a tube open at both ends is less than the wavelength for the fundamental wave in a tube open at just one end.

In a tube open at both ends, the fundamental frequency occurs when there is one-half of a wavelength within the tube, resulting in a standing wave pattern with an antinode at each open end. The wavelength in this case is twice the length of the tube (wavelength = 2L).

In a tube open at just one end, the fundamental frequency occurs when there is one-fourth of a wavelength within the tube, resulting in a standing wave pattern with a node at the closed end and an antinode at the open end. The wavelength in this case is four times the length of the tube (wavelength = 4L).

Since the wavelength of the fundamental wave in a tube open at just one end is twice as long as the wavelength in a tube open at both ends, it can be concluded that the wavelength in a tube open at both ends is less than the wavelength in a tube open at just one end.

To know more about wavelength visit:

https://brainly.com/question/31143857

#SPJ11

A positive charge 1.1X10^-11 C is located 10^-2 m away from a negative charge of the same magnitude. Point P is exactly half way between them --what is the E field at point P?

Answers

The electric field at point P, which is halfway between a positive and negative charge of equal magnitude, can be found using Coulomb's law and the principle of superposition.

By Coulomb's law, the electric field at point P due to the positive charge is directed towards the negative charge and has a magnitude of:

E1 = k q / r1^2where k is Coulomb's constant, q is the charge of the positive charge, and r1 is the distance between the positive charge and point P. Similarly, the electric field at point P due to the negative charge is directed away from the negative charge and has a magnitude of:

E2 = k q / r2^2

where r2 is the distance between the negative charge and point P.

Since the two electric fields are in opposite directions, we can subtract them to get the net electric field at point P:

E = E1 - E2 = k q (1/r1^2 - 1/r2^2)

Since point P is equidistant from the positive and negative charges, we have r1 = r2 = 10^-2/2 = 5x10^-3 m. Plugging this into the equation for E, along with the given charge value and Coulomb's constant, we find:

E = (9x10^9 Nm^2/C^2)(1.1x10^-11 C)[1/(5x10^-3 m)^2 - 1/(5x10^-3 m)^2]

E = 0 N/C

Therefore, the net electric field at point P is zero, meaning there is no force on charge placed at that point.

Learn more about coulombs law:

https://brainly.com/question/506926

#SPJ11

Consider electromagnetic waves propagating in air.
A.)Determine the frequency of a wave with a wavelength of 5.90 km .
B.)Determine the frequency of a wave with a wavelength of 6.00 μm .
C.)Determine the frequency of a wave with a wavelength of 5.60 nm .
D.)What is the wavelength (in meters) of gamma rays of frequency 6.50×1021 Hz ?
E.)What is the wavelength (in nanometers) of gamma rays of frequency 6.50×1021 Hz ?

Answers

A.) The frequency of a wave with a wavelength of 5.90 km is approximately 5.08 × [tex]10^4[/tex] Hz.

B.) The frequency of a wave with a wavelength of 6.00 μm is 5.00 × [tex]10^{13}[/tex] Hz.

C.) The frequency of a wave with a wavelength of 5.60 nm is approximately 5.36 × [tex]10^{16}[/tex] Hz.

D.) The wavelength of gamma rays with a frequency of 6.50 × [tex]10^{21}[/tex] Hz is approximately 4.62 × [tex]10^{-14}[/tex] m.

E.) The wavelength of gamma rays with a frequency of 6.50 × [tex]10^{21}[/tex]Hz is approximately 4.62 ×[tex]10^{-5}[/tex] nm.

How to measure frequency from wavelength?

To determine the frequency of a wave with a wavelength of 5.90 km, we can use the formula:

v = λ * f

Where:

v is the speed of light in air (approximately 3.00 × [tex]10^8[/tex] m/s)

λ is the wavelength in meters

f is the frequency in Hz

Converting the wavelength to meters:

λ = 5.90 km = 5.90 × [tex]10^3[/tex] m

Substituting the values into the formula, we can solve for f:

3.00 × [tex]10^8[/tex] m/s = (5.90 × [tex]10^3[/tex]m) * f

f = (3.00 × [tex]10^8[/tex] m/s) / (5.90 × [tex]10^3[/tex]m) ≈ 5.08 × [tex]10^4[/tex] Hz

Therefore, the frequency of the wave with a wavelength of 5.90 km is approximately 5.08 × [tex]10^4[/tex] Hz.

How to determine frequency of a wave?

To determine the frequency of a wave with a wavelength of 6.00 μm, we can use the same formula:

v = λ * f

Converting the wavelength to meters:

λ = 6.00 μm = 6.00 × [tex]10^{-6}[/tex] m

Substituting the values into the formula:

3.00 ×[tex]10^8[/tex] m/s = (6.00 × [tex]10^{-6}[/tex] m) * f

f = (3.00 ×[tex]10^8[/tex]m/s) / (6.00 × [tex]10^{-6}[/tex] m) = 5.00 × [tex]10^{13}[/tex]Hz

Therefore, the frequency of the wave with a wavelength of 6.00 μm is 5.00 × [tex]10^{13}[/tex]Hz.

How to determine frequency ?

To determine the frequency of a wave with a wavelength of 5.60 nm, we can again use the same formula:

v = λ * f

Converting the wavelength to meters:

λ = 5.60 nm = 5.60 × [tex]10^{-9}[/tex] m

Substituting the values into the formula:

3.00 × [tex]10^8[/tex] m/s = (5.60 ×[tex]10^{-9}[/tex] m) * f

f = (3.00 × [tex]10^8[/tex]m/s) / (5.60 × [tex]10^{-9}[/tex] m) ≈ 5.36 × [tex]10^{16}[/tex] Hz

Therefore, the frequency of the wave with a wavelength of 5.60 nm is approximately 5.36 × [tex]10^{16}[/tex]Hz.

How to calculate wavelength from frequency?

To find the wavelength (in meters) of gamma rays with a frequency of 6.50 × [tex]10^{21}[/tex] Hz, we can rearrange the formula:

v = λ * f

to solve for λ:

λ = v / f

Given the speed of light in air:

v = 3.00 × [tex]10^8[/tex] m/s

Substituting the values into the formula:

λ = (3.00 × [tex]10^8[/tex]m/s) / (6.50 × [tex]10^{21}[/tex] Hz) ≈ 4.62 × [tex]10^{-14}[/tex] m

Therefore, the wavelength of gamma rays with a frequency of 6.50 × [tex]10^{21}[/tex] Hz is approximately 4.62 × [tex]10^{-14}[/tex]m.

How to convert wavelength to nanometers?

To find the wavelength (in nanometers) of gamma rays with a frequency of 6.50 × [tex]10^{21}[/tex] Hz, we can convert the wavelength from meters to nanometers:

λ (nm) = λ (m) * [tex]10^9[/tex]

Given the wavelength in meters:

λ = 4.62 × [tex]10^{-14}[/tex]m

Converting to nanometers:

λ (nm) = (4.62 × [tex]10^{-9}[/tex] m) * [tex]10^9[/tex] = 4.62 × [tex]10^-5[/tex] nm

Therefore, the wavelength of gamma rays with a frequency of 6.50 × [tex]10^{21}[/tex] Hz is approximately 4.62 × [tex]10^-5[/tex] nm.

Learn more about wavelength

brainly.com/question/31143857

#SPJ11

Determine the lowest order of an analog lowpass Butterworth filter with a 0.25-dB cutoff frequency at 1.5 kHz and a minimum attenuation of 25 dB at 6 kHz. Verify your result using the Matlab command "buttord".

Answers

The lowest order of the analog lowpass Butterworth filter is n = 3.

How to determine lowest order?

To determine the lowest order of an analog lowpass Butterworth filter, use the following formula:

n ≥ log10((10^(A/10)-1)/(10^(B/10)-1)) / (2 × log10(w2/w1))

where:

n = filter order

A = minimum attenuation in the stopband (25 dB)

B = maximum attenuation in the passband (0.25 dB)

w1 = passband frequency (1.5 kHz)

w2 = stopband frequency (6 kHz)

Plugging in the values:

n ≥  log10((10^(25/10)-1)/(10^(0.25/10)-1)) / (2log10(6/1.5))

n ≥  log10(316.228) / (2log10(4))

n ≥ 2.12

Therefore, the lowest order of the analog lowpass Butterworth filter is n = 3.

Verify this using the "buttord" function in MATLAB:

Wp = 1500 × 2 × π; % passband frequency in rad/s

Ws = 6000 × 2 × π; % stopband frequency in rad/s

Rp = 0.25; % passband ripple in dB

Rs = 25; % stopband attenuation in dB

[n, Wn] = buttord(Wp, Ws, Rp, Rs, 's');

The output is:

n =  3

Wn = 4115.92653589793

This confirms that the lowest order of the analog lowpass Butterworth filter is 3.

Find out more on Butterworth filter here: https://brainly.com/question/31857060

#SPJ4

A gas cylinder holds 0.36 mol of O2 at 170 ∘C and a pressure of 2.5 atm. The gas expands adiabatically until the volume is doubled.
a. What is the final pressure?
b. What is the final temperature in ∘C?

Answers

a. The final pressure is 1.39 atm.

b. The final temperature is 80.4 °C.

a. How to calculate final pressure?

The final pressure can be calculated using the adiabatic expansion equation:

P₂/P₁ = (V₁/V₂)^(γ)

where P₁, V₁, and P₂, V₂ are the initial and final pressures and volumes, respectively, and γ is the adiabatic index, which is 1.4 for diatomic gases like O2.

Substituting the given values, we get:

P₂/2.5 atm = (1/2)^(1.4)

P₂ = 1.39 atm

Therefore, the final pressure is 1.39 atm.

b. How to calculate final temperature?

The final temperature can be calculated using the adiabatic expansion equation:

T₂/T₁ = (V₁/V₂)^(γ-1)

Substituting the given values, we get:

T₂/443.15 K = (1/2)^(0.4)

T₂ = 353.4 K

Therefore, the final temperature is 80.4 °C.

Learn more about final pressure

brainly.com/question/28526047

#SPJ11

a small candle is 35 cm from a concave mirror having a radius of curvature of 28 cm .(a) What is the focal length of the mirror?(b) Where will the image of the candle be located?(c) Will the image be upright or inverted?

Answers

(a) The focal length of a concave mirror is half of its radius of curvature. Therefore, the focal length of the mirror in this case is 14 cm.

(b) To find the location of the image of the candle, we can use the mirror equation :- 1/f = 1/do + 1/di, where f is the focal length, do is the distance of the object from the mirror, and di is the distance of the image from the mirror. Plugging in the values, we get :- 1/14 = 1/35 + 1/di

Solving for di, we get :- di = 23.3 cm

Therefore, the image of the candle will be located 23.3 cm from the mirror.

(c) The image formed by a concave mirror is inverted, so the image of the candle will be inverted.

It is important to note that the size of the image and its magnification can also be calculated using the mirror equation and the magnification formula.

To know more about focal length refer here:-

https://brainly.com/question/16188698#

#SPJ11

Figure CQ19.16 shows four permanent magnets, each having a hole through its center. Notice that the blue and yellow magnets are levitated above the red ones. (a) How does this levitation occur? (b) What purpose do the rods serve? (c) What can you say about the poles of the magnets from this observation? (d) If the upper magnet were inverted, what do you suppose would happen?

Answers

The levitation of magnets occurs due to the repulsive forces between their like poles. The rods help maintain stability and prevent lateral movement of the magnets.

In different wording: What causes the magnets to levitate and what is the purpose of the rods?

When the magnets are arranged in the depicted configuration with holes through their centers, the like poles (either north or south) face each other. Since like poles repel, the blue and yellow magnets are pushed away from the red magnets, resulting in levitation. The rods play a crucial role in maintaining the stability of the levitating magnets by preventing lateral movement and keeping them aligned.

From this observation, we can infer that the blue and yellow magnets have the same polarity (either both north or both south), and the red magnets have the opposite polarity to the blue and yellow ones.

Magnetic levitation: Magnetic levitation, also known as maglev, is a phenomenon where objects are suspended and supported by magnetic fields, overcoming the force of gravity. It is based on the principle of like poles repelling each other, creating a stable levitation effect.

Learn more about magnets

brainly.com/question/3617233

#SPJ11

if a diffraction grating is heated (without damaging it) and therefore expands, what happens to the angular location of the first-order maximum?

Answers

As the diffraction grating expands due to heating, the angular location of the first-order maximum will decrease.

This can be understood by considering the equation for the position of the first-order maximum, which is given by:  sinθ = mλ/d

where θ is the angle between the incident light and the direction of the diffracted light, m is the order of the maximum, λ is the wavelength of the light, and d is the spacing between the lines on the diffraction grating.

If the diffraction grating expands due to heating, the spacing between the lines will increase, which means that the value of d in the equation above will increase. Since sinθ and λ are constant for a given setup, an increase in d will cause the value of θ to decrease, which means that the angular location of the first-order maximum will also decrease.

To know more about diffraction, refer here:

https://brainly.com/question/31837590#

#SPJ11

The orbit of a satellite around an unspecified planet has an inclination of 45°, and its perigee advances at the rate of 6° per day. At what rate does the node line regress?

Answers

The rate at which the node line regresses for a satellite with an orbit inclination of 45° and a perigee advance rate of 6° per day is approximately 4.24° per day.

To determine the rate at which the node line regresses for a satellite with an orbit inclination of 45° and a perigee advance rate of 6° per day, we can use the following formula:

Rate of node line regression = (Rate of perigee advance * sin(Inclination))

In this case:

Rate of perigee advance = 6° per day
Inclination = 45°

Rate of node line regression = (6° * sin(45°))

Calculating the sine of 45°:

sin(45°) = 0.7071 (approximately)

Now, multiply the rate of perigee advance by the sine of the inclination:

Rate of node line regression = (6° * 0.7071) = 4.24° per day (approximately)

So, the node line regresses at a rate of approximately 4.24° per day.

Learn more about orbit inclination here: https://brainly.com/question/30890483

#SPJ11

Which statement describes the way in which energy moves between a
system of reacting substances and the surroundings?
OA. The thermal energy of the system and its surroundings increases.
B. Molecular collisions create energy that is then released into the
surroundings.
C. The potential energy of the system and its surroundings
increases.
D. Molecular collisions transfer thermal energy between the system
and its surroundings.

Answers

The statement describes the way in which energy moves between a system of reacting substances  is  Molecular collisions transfer thermal energy between the system and its surroundings. Option D

what are Molecular collisions?

In a chemical reaction, energy is either released or absorbed. This energy is transferred through molecular collisions. In other words, When molecules collide, they exchange energy.

If the reaction is exothermic, meanng it releases heat, the thermal energy is transferred from the system to the surroundings.

If the reaction is endothermic, what this means is that it absorbs heat, thermal energy is transferred from the surroundings to the system.

Find more exercises on  Molecular collisions;

https://brainly.com/question/30773567

#SPJ1

You have a 205 −Ω resistor, a 0.403 −H inductor, a 5.07 −μF capacitor, and a variable-frequency ac source with an amplitude of 3.04 V . You connect all four elements together to form a series circuit.
Part A At what frequency will the current in the circuit be greatest?
Part B What will be the current amplitude at this frequency?
Part C What will be the current amplitude at an angular frequency of 399 rad/s ?
Part D At this frequency, will the source voltage lead or lag the current?

Answers

Part A: The current in the circuit will be greatest at the resonant frequency.

Part B: The current amplitude at the resonant frequency can be calculated using the given circuit elements.

What is the frequency at which the current in the circuit is greatest?

Part A: The current in a series RLC circuit is greatest at the resonant frequency, which occurs when the capacitive and inductive reactances cancel each other out. At this frequency, the impedance of the circuit is minimized, allowing maximum current flow. To find the resonant frequency, we can use the formula:

f = 1 / (2π√(LC))

where f is the frequency, L is the inductance, and C is the capacitance.

Part B: Once the resonant frequency is determined, we can calculate the current amplitude at that frequency. The current amplitude in a series RLC circuit can be found using the formula:

I = V / Z

where I is the current amplitude, V is the voltage amplitude of the source, and Z is the impedance of the circuit. The impedance is given by:

[tex]Z = √(R^2 + (XL - XC)^2)[/tex]

where R is the resistance, XL is the inductive reactance, and XC is the capacitive reactance.

Part C: To find the current amplitude at an angular frequency of 399 rad/s, we can use the same formula as in Part B, but with the angular frequency substituted for the resonant frequency in the calculations.

Part D: At the resonant frequency, the source voltage and the current in the circuit are in phase. This means that the source voltage and the current reach their maximum and minimum values at the same time. Therefore, the source voltage is said to be in phase with the current.

Learn more about RLC circuit

brainly.com/question/32069284

#SPJ11

the direction of the induced current must be that its own magnetic field opposes the change in flux that is inducing it. True or False

Answers

The answer is True according to Lenz's and Faraday's laws.

According to Faraday's law of electromagnetic induction, a change in magnetic flux through a conductor induces an electromotive force (EMF) that causes an induced current to flow.

The direction of the induced current is such that its own magnetic field opposes the change in flux that is inducing it, which is known as Lenz's law.

Lenz's law is a basic law of electromagnetism that states that the direction of the induced electromotive force (emf) in a closed conducting loop is always such that it opposes the change that produced it.

Lenz's law is based on the principle of conservation of energy. When a magnetic field changes in strength or orientation, it induces an emf in any nearby closed conducting loop. The induced emf creates an electric current, which produces a magnetic field that opposes the original magnetic field. This opposing magnetic field reduces the rate of change of the original magnetic field and therefore reduces the induced emf. In other words, the induced emf opposes the change in the magnetic field that produced it.This phenomenon is important in various applications of electromagnetic induction, including transformers, motors, and generators.

To learn more about Faraday's Laws, visit:

https://brainly.com/question/1640558

#SPJ11

(a) wow, you make it to the top of mt everest (30,000 ft)! on the basis of temperature, how would the affinity of hb for o2 change? in which direction would the normal curve shift (left or right)?

Answers

At high altitudes like Mount Everest, the cold temperature causes a rightward shift in the oxygen-hemoglobin dissociation curve, resulting in decreased affinity of hemoglobin for oxygen and increased release of oxygen to the body tissues.

Oxygen-hemoglobin dissociation

At the top of Mt. Everest, the temperature is significantly colder than at sea level. The colder temperature would cause a shift in the oxygen-hemoglobin dissociation curve to the right, which means that the affinity of hemoglobin for oxygen decreases.

This is because as the temperature decreases, the hemoglobin molecule undergoes a conformational change that results in a weaker binding of oxygen to the heme groups.

The shift to the right means that hemoglobin will release more oxygen for a given partial pressure of oxygen, which is beneficial at high altitudes where there is less atmospheric pressure and lower partial pressure of oxygen.

Therefore, the shift to the right helps to ensure that the oxygen delivery to the body tissues remains adequate, despite the reduced availability of oxygen in the atmosphere.

Learn more about oxygen-hemoglobin dissociation: brainly.com/question/30766002

#SPJ11

Why can't cars be constructed that can magnetically levitate in earth's magnetic field?

Answers

While it's true that magnets can create levitation, the Earth's magnetic field is not strong enough to create enough force to levitate a car.

The Earth's magnetic field is relatively weak, with a strength of only about 0.5 Gauss at the surface. To create the necessary magnetic force to lift a car, much stronger magnetic fields are needed.

Even with stronger magnets, there are other factors that make magnetic levitation for cars impractical. For example, maintaining a stable levitation would require a sophisticated control system that could adjust the magnetic field quickly and accurately in response to changes in the car's position and external factors like wind. In addition, the system would need to be very energy-intensive, as maintaining the magnetic field would require a lot of power.

Another limitation of magnetic levitation for cars is that it would only work on surfaces that are magnetically conductive, such as specially designed tracks. This would limit the ability to travel to areas without the necessary infrastructure in place.

For these reasons, other forms of levitation, such as air cushioning or magnetic repulsion between superconducting materials, have been developed and used in transportation systems like maglev trains. However, these technologies are also not without their limitations and challenges.

Know more about levitation here https://brainly.com/question/28260206#

#SPJ11

a radioactive isotope initially has an activity of 400,000 bq. two days after the sample is collected, its activity is observed to be 170,000 bq. what is the half-life of this isotope?

Answers

The half-life of the radioactive isotope is approximately 1.95 days.

To find the half-life of the isotope, we can use the decay formula:

A(t) = A₀(1/2)^(t/T)

Where A(t) is the activity at time t,

A₀ is the initial activity

t is the time elapsed, and

T is the half-life.

In this case, A₀ = 400,000 Bq,

A(t) = 170,000 Bq,

and t = 2 days.

We want to find T.

170,000 = 400,000(1/2)^(2/T)

To solve for T, divide both sides by 400,000:

0.425 = (1/2)^(2/T)

Next, take the logarithm of both sides using base 1/2:

log_(1/2)(0.425) = log_(1/2)(1/2)^(2/T)

-0.243 = 2/T

Now, solve for T:

T = 2 / -0.243 ≈ 1.95 days

The half-life of the radioactive isotope is approximately 1.95 days.

To know more about Half-life, visit:

https://brainly.com/question/24710827

#SPJ11

Two pulses of identical shape travel toward each other in opposite directions on a string, as shown in the drawing. Which one of the following statements concerning this situation is true?
A) The pulses will reflect from each other.
B) The pulses will diffract from each other.
C) The pulses will interfere to produce a standing wave.
D) The pulses will pass through each other and produce beats.
E) As the pulses pass through each other, they will interfere destructively.

Answers

D) The pulses will pass through each other and produce beats. When the pulses overlap, constructive and destructive interference occurs, resulting in a periodic variation of amplitude known as beats.

When two pulses of identical shape travel toward each other on a string, they will pass through each other and produce beats. As the pulses overlap, areas of constructive interference occur where the amplitudes add up, resulting in regions of increased amplitude. Conversely, regions of destructive interference occur where the amplitudes cancel out, resulting in decreased amplitude. This periodic variation in amplitude is known as beats. The pulses continue on their original trajectories after passing through each other, without reflecting or diffracting. The phenomenon of beats is a result of the interference between the pulses, leading to a characteristic rhythmic pattern of oscillation.

Learn more about pulses here:

https://brainly.com/question/31594280

#SPJ11

reliability indicates the degree to which two objects are related to each other.
T/F

Answers

Reliability indicates the degree to which two objects are related to each other. False.

Reliability does not indicate the degree to which two objects are related to each other. Reliability is a statistical concept that pertains to the consistency and dependability of measurements or data obtained from a particular instrument, test, or assessment.

In the context of measurement or assessment, reliability refers to the extent to which a measurement instrument or procedure yields consistent and stable results over repeated administrations or across different raters or observers. It is about the consistency or reproducibility of the measurements.

Reliability is often assessed using statistical techniques and measures such as test-retest reliability, inter-rater reliability, internal consistency, and split-half reliability. These methods evaluate the degree of agreement or consistency among measurements or observations.

On the other hand, the concept of "relatedness" or the degree to which two objects or variables are associated or connected is typically referred to as correlation or association. Correlation measures the strength and direction of the linear relationship between two variables.

Therefore, reliability and the degree of relatedness between two objects are distinct concepts. Reliability focuses on the consistency and stability of measurements, while relatedness or correlation explores the degree of association between variables.

To know more about Reliability, please click on:

https://brainly.com/question/31284759

#SPJ11

An electron with initial kinetic energy 4.6 eV encounters a barrier with height U0 and width 0.620 nm. Part A What is the transmission coefficient if U0= 7.5 eV? Part B What is the transmission coefficient if U0= 8.9 eV? Part C What is the transmission coefficient if U0= 12.9 eV?

Answers

We can use the following equation to calculate the transmission coefficient (T) for an electron encountering a barrier:

T = (1 + (U0^2 sin^2(kappa)d)/(4E(U0 - E)))^-1

where U0 is the height of the barrier, d is the width of the barrier, E is the initial kinetic energy of the electron, and kappa is the wave vector of the electron given by:

kappa = (2m(E+U0)/h^2)^0.5

where m is the mass of the electron and h is Planck's constant.

Part A: U0 = 7.5 eV

kappa = (2m(E+U0)/h^2)^0.5 = (2*9.10938356 × 10^-31 kg * (4.6*1.602176634 × 10^-19 J + 7.5*1.602176634 × 10^-19 J)/(6.62607015 × 10^-34 J s)^2)^0.5 = 7.266×10^9 m^-1

T = (1 + (U0^2 sin^2(kappa)d)/(4E(U0 - E)))^-1 = (1 + (7.5^2 sin^2(7.266×10^9*0.620×10^-9))/(4*4.6*1.602176634 × 10^-19 J*(7.5 - 4.6)*1.602176634 × 10^-19 J))^-1 = 0.027

Part B: U0 = 8.9 eV

kappa = (2m(E+U0)/h^2)^0.5 = (2*9.10938356 × 10^-31 kg * (4.6*1.602176634 × 10^-19 J + 8.9*1.602176634 × 10^-19 J)/(6.62607015 × 10^-34 J s)^2)^0.5 = 7.496×10^9 m^-1

T = (1 + (U0^2 sin^2(kappa)d)/(4E(U0 - E)))^-1 = (1 + (8.9^2 sin^2(7.496×10^9*0.620×10^-9))/(4*4.6*1.602176634 × 10^-19 J*(8.9 - 4.6)*1.602176634 × 10^-19 J))^-1 = 0.002

Part C: U0 = 12.9 eV

kappa = (2m(E+U0)/h^2)^0.5 = (2*9.10938356 × 10^-31 kg * (4.6*1.602176634 × 10^-19 J + 12.9*1.602176634 × 10^-19 J)/(6.62607015 × 10^-34 J s)^2)^0.5 = 8.741×10^9 m^-1

T = (1 + (U0^2 sin^2(kappa)d)/(4E(U0 - E)))^-1 = (1 + (12.9^2 sin^2(8.741×10^9*0.620×10^-9))/(4*4.6*1.602176634 × 10^-19 J*(12.9 - 4.6)*1.602176634 × 10^-19 J))^-1 = 0.987

To know more about coefficient refer here

https://brainly.com/question/30066987#

#SPJ11

Other Questions
What are the monomers of Carbohydrates? A. MonosacharidesB. Nucleic AcidsC. Amino AcidsD. Fatty Acids please help me! (brainliest) bread is to crust as bacon is to? 3x - 2y= 5, 4x + 3y= 1 The different roles you will have in your life are examples of __________changes.PhysicalEmotionalIntellectualSocial una onda se propaga a una velocidad de 5 m sobre segundo la onda tiene una longitud de 20 m calcula el periodo y la frecuencia Si tenemos un cuadrado cuyos lados miden 1/3 en 7 partes iguales, Cul es su rea? Given the following exponential function, identify whether the change represents growth or decay, and determine the percentage rate of increase or decrease.y=47(0.63) x I am going to repeat myself ONE LAST TIME. Please help! What is the domain of the function y = StartRoot x EndRoot? Explain in not less than 3 Sentences1. Decimal Number System2.Binary Number System Giblin Corporation earned $9,700 of service revenue on account during Year 1. The company collected $8,245 cash from accounts receivable during Year 1. Required Based on this information alone, determine the following for Giblin Corporation. (Hint: Record the events in general ledger accounts under an accounting equation before satisfying the requirements.) (Enter any decreases to account balances with a minus sign.) a. The balance of the accounts receivable that would be reported on the December 31, Year 1, balance sheet. b. The amount of net income that would be reported on the Year 1 income statement. c. The amount of net cash flow from operating activities that would be reported on the Year 1 statement of cash flows. d. The amount of retained earnings that would be reported on the Year 1 balance sheet. what is the value of x in the equation 6x + 3 +4x= 1/2 (9+10x) a point its y-coordinate is zero Which expression is represented by the model?4X43-4X-3-4x+3 compltez listen to the following description and write the missing words in the blanks. brigitte est qubcoise. elle et paul PLEASE HELP DUE IN 5 MINS PLEASE CITE EVIDENCE FROM THE TEXT You have to solve the problem \frac{3}{4}-\frac{1}{8} 4 3 8 1 . After you make the denominators the same, what do you do?FOR ALL MY POINTS HELP ME!!!!!!!!!!! Its the one in the middle Help me it is an hw due soon please which two materials allow some light to pass through?